phlee004
Thanks Received: 0
Forum Guests
 
Posts: 6
Joined: May 14th, 2012
 
 
 

Q16 - Any member of the city council

by phlee004 Wed May 23, 2012 6:25 am

Hey guys,

Although I understand why B is correct, I'm having trouble eliminating A. Any suggestions?

Thank you =)
 
giladedelman
Thanks Received: 833
LSAT Geek
 
Posts: 619
Joined: April 04th, 2010
 
This post thanked 1 time.
 
 

Re: Q16 - Any member of the city council

by giladedelman Mon May 28, 2012 2:11 pm

Thanks for the question!

So we've got two premises: 1) everyone on the city council should either vote "no" or abstain, and 2) if they all abstain, the issue will be decided by the voters.

From these premises, the argument concludes that at least one member should vote "no." I add emphasis to the word "should" because it's such an important word in any argument, especially in the conclusion. How do we know that at least one council member "should" vote, as opposed to everyone abstaining? Well, all we know about the latter case is that the issue would get decided by the voters ... so we have to be assuming that the issue should not be decided by the voters.

That's why (B) is correct.

The problem with (A) is that even if we make this assumption, we still have no basis for concluding that at least one person on the council should vote against the proposal. Although the premises say that council members should vote "no" if they vote, nowhere do they say that the proposal is bad in and of itself. So just because the voters would approve it doesn't mean the council should vote. We're still missing the assumption that having the voters vote on the issue (in whichever way) is something we want to avoid.

(C) is incorrect because how the council members actually vote has nothing to do with whether they should vote or abstain.

(D) is a negation of the second premise, and doesn't help us decide whether the voters should get to decide or not.

(E) is incorrect because we don't need to assume that everyone else should abstain; the argument leaves open the possibility that everyone could vote.

Does that clear this one up for you?
 
jawu9
Thanks Received: 0
Forum Guests
 
Posts: 1
Joined: September 29th, 2012
 
 
 

Re: Q16 - Any member of the city council

by jawu9 Sat Sep 29, 2012 9:54 pm

I think D can be correct here!

I interpreted "But if all the members abstain, the matter will be decided by the city's voters" as an if --> then statement. We know that all abstentions will cause the voters to decide. But the stimulus never explicitly states that at least one council vote will prevent the proposal from being decided by voters. What if the matter will be decided by voters even if some members vote? Answer choice D would bridge that assumption.

The stimulus seems to imply that the voters will not decide if at least one vote is cast, but there is not enough information to infer that!

I hope I'm being clear here.
User avatar
 
rinagoldfield
Thanks Received: 308
Atticus Finch
Atticus Finch
 
Posts: 390
Joined: December 13th, 2011
 
This post thanked 2 times.
 
 

Re: Q16 - Any member of the city council

by rinagoldfield Mon Oct 01, 2012 4:25 pm

Hello! I developed an in-depth solution to this problem during my curriculum training to become an instructor with Manhattan LSAT. Here's my take on things.
Attachments
PT65, S1, Q16 - Manhattan LSAT - Any member of the city council ought.pdf
(81 KiB) Downloaded 2053 times
 
aznriceboi17
Thanks Received: 5
Elle Woods
Elle Woods
 
Posts: 76
Joined: August 05th, 2013
 
 
trophy
Most Thankful
 

Re: Q16 - Any member of the city council

by aznriceboi17 Sun Sep 22, 2013 7:20 pm

Hi, I had the same question as jawu9. I've read the explanation that rinagoldfield provided, however I don't understand how D is negates the logic of the second premise.

My understanding was the same as jawu9's: the second premise only says what happens if everyone abstains -- the city's voters then decide the matter. The stimulus does not actually state that if we get at least one non-abstaining vote then the matter will not be decided by the voters -- something that D seems to provide.

Am I missing something there? Thanks!
 
jwms
Thanks Received: 0
Jackie Chiles
Jackie Chiles
 
Posts: 30
Joined: October 16th, 2014
 
 
 

Re: Q16 - Any member of the city council

by jwms Thu Feb 05, 2015 1:58 pm

I think *should* is pretty key here. (D) is a statement of fact that doesn't address the should statement in the Conclusion.

I wonder, on SA Qs, if 'should' is in the Conclusion, if it will most likely be in TCR?
User avatar
 
rinagoldfield
Thanks Received: 308
Atticus Finch
Atticus Finch
 
Posts: 390
Joined: December 13th, 2011
 
This post thanked 1 time.
 
 

Re: Q16 - Any member of the city council

by rinagoldfield Fri Feb 06, 2015 3:14 pm

Thanks aznriceboi17 and jwms !
Great questions.

Here’s my summary of the argument:

The author sees two paths.
Path 1: everyone abstains
Path 2: someone votes no
Path 1 leads to city voters.
Therefore, let’s take Path 2.

Aznriceboi17, I think you are saying that we need to know that Path 2 will take the issue away from city voters. After all, the author wants to avoid the path that would lead to the city voters. Isn’t it necessary to know that the alternative path avoids the voters?

No. We just need to know that we should avoid the city voters. If we know that we should avoid city voters, then we know we need to avoid Path 1, since Path 1 guarantees the city voters.

Here is an analogy for the argument:

I can stay in high school or drop out of high school. Dropping out means I will not go to college. Therefore, I should stay in high school.

Path 1: drop out
Path 2: stay in school
Path 1 leads to no college.
Therefore we should take Path 2.

The assumption is not that high school guarantees college. Rather, the assumption here is that I should go to college. If we know that I should go to college, then we know we shouldn’t take Path 1, which guarantees no college.

An analogous answer choice (D) would say:

If I stay in high school, I will go to college

An analogous answer choice (B), the correct answer says:

I should go to college.

(B) is correct.

***

By the by, (D) does negate the second premise.
Premise 2: If all abstain, then city voters.
(D): If not all abstain, then not city voters.

Noticing this can make the elimination of (D) a little easier – the questions you guys are bringing up are smart, but (D) really just twists around a premise.

Hope this helps.
 
ying_yingjj
Thanks Received: 1
Jackie Chiles
Jackie Chiles
 
Posts: 28
Joined: March 12th, 2014
 
 
 

Re: Q16 - Any member of the city council

by ying_yingjj Wed May 20, 2015 10:50 pm

this is an example of LSAC's poor question drafting. The question itself has so many issues.

(A) should be the perfect answer explaining why at least one member should vote against.

(B) is just contrapositive the chain from the premise: if all abstain (-VA) ---> city voters vote
conclusion: at least 1 VA (-abstain).

if we contra : if all abstain (-VA) ---> city voters vote

we get: -city voters vote ----> VA (all VA or at least 1 VA)

So, "city voters not vote" is not an assumption. Assumption is something hidden, assumed by the author. but "city voters not vote" is merely the contra of the premise.

If I say, If all students wanting to go to law school believe LSAT score is so important ----> LSAC will continue to make lame questions. Then, my conclusion says, We should not believe LSAT score .

What's my assumption? "We should not let LSAC continue making lame questions" is not my assumption, it is merely a contrapositive of my premise.
 
donghai819
Thanks Received: 7
Elle Woods
Elle Woods
 
Posts: 65
Joined: September 25th, 2015
 
 
 

Re: Q16 - Any member of the city council

by donghai819 Mon Jan 25, 2016 3:21 pm

There is an unique pattern right here. I can vaguely recall there is another very very similar question about comet hitting planet or some similar staff. Very interesting.
 
dhlim3
Thanks Received: 4
Jackie Chiles
Jackie Chiles
 
Posts: 34
Joined: January 19th, 2015
 
 
 

Re: Q16 - Any member of the city council

by dhlim3 Mon Apr 18, 2016 3:43 am

The answer makes much more sense if you only consider the reasoning side of the argument.

For simplicity, let's use letters.

A = Council Vote Against
B = Council Abstain
C = City's Voters Decide

Premise #1: A <==> ~B (Either Vote Against or Abstain)
Premise #2: B ==> C (If Abstain, then City's Voters Decide)
Assumption: ??????
Conclusion: A (Vote Against)

The correct answer will state a condition that will sufficiently lead to A. Based on the premises given, the only way to get to A is by negating B (~B ==> A). But what would make it necessary to negate B? By negating C (~C ==> ~B, which is the contrapositive of Premise #2). So the correct answer will state that C should not happen (City's Voters must NOT Decide).

So with the missing chain resolved:

Premise 1: A <==> ~B (Either A or B)
Premise 2: B ==> C (If B, then C)
Assumption: ~C (C should not happen)
Conclusion: ~C ==> ~B ==> A

Answer B correctly states this missing link word for word.



Answer A, on the other hand, introduces a new component to the reasoning (The proposal will be passed).

The diagram will looks like this.

A = Council Vote Against
B = Council Abstain
C = City's Voters Decide
D = Proposal Passed

Premise #1: A <==> ~B (Either Vote Against or Abstain)
Premise #2: B ==> C (If Abstain, then City's Voters Decide)
Using Answer A: B ==> D
Conclusion: A (Vote Against)

You can see from the above diagram that answer A does nothing to fill the missing gap. In fact, it further complicates the reasoning by introducing a new component.

If Answer A is to be correct, then we would need an additional premise stating that the Proposal should not be passed. But the stimulus never states this. It only states that Council should either vote against or abstain from voting, but it is logically insufficient to take this premise and infer that the activist does not want the proposal passed.
 
layamaheshwari
Thanks Received: 5
Vinny Gambini
Vinny Gambini
 
Posts: 22
Joined: April 23rd, 2016
 
 
 

Re: Q16 - Any member of the city council

by layamaheshwari Thu Jun 16, 2016 7:41 am

I got this question right, and used a tip that Mary Adkins had posted on the thread for another sufficient question.

Simply put: the premises are all facts (variations of "is"), while the conclusion is a recommendation ("should" is key). Since this is a sufficient assumption question, our answer choice will have to guarantee the conclusion when plugged in to the stimulus and thus must be responsible for jump from facts to recommendation, i.e. it should also have "should" or some variation in it. I selected B because of that, and it was right!

Is this strategy foolproof, or am I setting myself up for a fall?
 
erikwoodward10
Thanks Received: 9
Elle Woods
Elle Woods
 
Posts: 69
Joined: January 26th, 2014
 
 
 

Re: Q16 - Any member of the city council

by erikwoodward10 Mon Jul 25, 2016 4:11 pm

layamaheshwari Wrote:I got this question right, and used a tip that Mary Adkins had posted on the thread for another sufficient question.

Simply put: the premises are all facts (variations of "is"), while the conclusion is a recommendation ("should" is key). Since this is a sufficient assumption question, our answer choice will have to guarantee the conclusion when plugged in to the stimulus and thus must be responsible for jump from facts to recommendation, i.e. it should also have "should" or some variation in it. I selected B because of that, and it was right!

Is this strategy foolproof, or am I setting myself up for a fall?

Yup. When the conclusion is qualified, we must have a qualifier in the premises in order for the conclusion to be valid. How can we conclude that something "should" happen if we don't have any qualified premises? We can't! This is a pretty simple question for exactly this reason, all of the other answer choices don't tell us why something *SHOULD* happen.
 
andrewgong01
Thanks Received: 61
Atticus Finch
Atticus Finch
 
Posts: 289
Joined: October 31st, 2016
 
 
 

Re: Q16 - Any member of the city council

by andrewgong01 Sun Aug 13, 2017 1:41 am

erikwoodward10 Wrote:
layamaheshwari Wrote:I got this question right, and used a tip that Mary Adkins had posted on the thread for another sufficient question.

Simply put: the premises are all facts (variations of "is"), while the conclusion is a recommendation ("should" is key). Since this is a sufficient assumption question, our answer choice will have to guarantee the conclusion when plugged in to the stimulus and thus must be responsible for jump from facts to recommendation, i.e. it should also have "should" or some variation in it. I selected B because of that, and it was right!

Is this strategy foolproof, or am I setting myself up for a fall?

Yup. When the conclusion is qualified, we must have a qualifier in the premises in order for the conclusion to be valid. How can we conclude that something "should" happen if we don't have any qualified premises? We can't! This is a pretty simple question for exactly this reason, all of the other answer choices don't tell us why something *SHOULD* happen.



What if we had the additional premise like " This bill is bad" [NOT This bill must not be/ should not be] pass]? Wouldn't this allow "A" to be a viable choice as a SA if we didn't have Choice "B"? Then "A" could still work without a "should" in the answer choice because now we are making a tiny inference that the LSAC/LSAT allows that "Bad" is "bad" and we should avoid "bad" outcomes
User avatar
 
ohthatpatrick
Thanks Received: 3805
Atticus Finch
Atticus Finch
 
Posts: 4661
Joined: April 01st, 2011
 
 
 

Re: Q16 - Any member of the city council

by ohthatpatrick Wed Aug 16, 2017 3:16 pm

Yes, if we already knew that the bill's passage was an outcome we SHOULD avoid, then (A) would work. (Of course it also involves an assumption that "if the voters decide in favor of the proposal, the proposal will be enacted" ... a majority of votes doesn't necessarily mean a victory, as the US Presidential Elections sometimes remind us)

LSAT is often fine with switching from "bad" to "should not", but I wouldn't expect to see that on something so verbatim and literal as Sufficient Assumption.